Q4

 
mcbrown.iv
Thanks Received: 0
Vinny Gambini
Vinny Gambini
 
Posts: 1
Joined: November 28th, 2010
 
 
 

Q4

by mcbrown.iv Tue Aug 02, 2011 4:31 pm

Good afternoon,

I was wondering if someone could please explain Question #4. I diagrammed the problem correctly, but was wondering if there was a specific strategy used for attacking this question other than trying to diagram each answer choice one by one?

Thanks
 
timmydoeslsat
Thanks Received: 887
Atticus Finch
Atticus Finch
 
Posts: 1136
Joined: June 20th, 2011
 
This post thanked 4 times.
 
trophy
Most Thanked
trophy
First Responder
 

Re: Q4

by timmydoeslsat Tue Aug 02, 2011 6:02 pm

There is a strategy other than diagramming each choice!

It can be a big time saver.

We know the rules are:

M (3) ---> L (6)

Z (1) ---> L > O

T,K = ~ 5 (~ = not)

The last rule can be seen as this: T/K - M - K/T

We know that M cannot come before both of them, but we do know it is one of them, thus we can deduce one is before and one is after, as for which goes where, we do not know at this point.

Question 4 asks us for a partial lineup question. They give us only two selections out of 6.

A) L: 2 M: 3
B) M: 4 L: 6
C) O: 2 M: 5
D) Z: 1 L: 6
E) Z: 1 M: 2

I would look for the easiest answer choices to eliminate at this point.

We have a constraint concerning what happens when Z is (1).

Choices D and E are relevant to our interests right now. When Z is (1), we know that due to the second rule, that L comes before O. Since there are only six slots available, there is no way for L to be sixth, as O has to come after it, thus we know that choice D can be eliminated!

Choice E is now up for discussion. Z being first in this case is not giving us dramatic consequences, we know that L has to come before O, but this choice does not implicate anything being wrong. However, do look at who is occupying the second slot.

M is.

What do we know about this M variable? We know that one of T and K comes before it. In this answer choice, the only variable before it is Z. Thus, we know that both K and T will be AFTER M in this scenario. Eliminate!

We are down to A, B, and C.

Let me bring these choices down for review for us:

A) L: 2 M: 3
B) M: 4 L: 6
C) O: 2 M: 5

The next most enticing answer choice I see is (A). I am seeing M in the third spot. What do I know about M being third? I know that when it is third, L must be 6th. This is because o the first rule. In answer choice A, L is second and not sixth. Eliminate!

B) M: 4 L: 6
C) O: 2 M: 5

At this point you could decide to do a hypothetical for just one of them to test it out. If it works, select it. If it does not work, select the other, assuming you executed it correctly.

However, a savvy way of doing this is to consider the placement of M. We know that we must have one of T and K after it and before it. With answer choice (B), we see that M is in the fourth position. We know that one of the two remaining spaces of the 6 slots will have to contain one of T and K for this to be a valid scenario.

However, we know that T and K are not fifth due to the third rule. This means that they must go 6th. But L is in the 6th slot! No room for one of T and K to go after M! Eliminate!

We are left with answer choice C. No need to even check it. Move on!